What is the maximum value of $x^TAx$ subject to $xin{pm1}^n$?











up vote
2
down vote

favorite












Let $A in mathbb{R}^{ntimes n}$ be symmetric and positive definite. What is the following maximum?



$$max_{xin{pm1}^n}x^T A x$$





My attempt:



if all $a_{ij}geq 0$, then



begin{equation}
max_{xin{pm1}^n}x^TAx=sum a_{ij}
end{equation}



and in this case $x=[1quad 1quadcdotsquad 1]'$ or $x=[-1quad -1quadcdotsquad -1]'$.



if some $a_{ij}<0$, then situation is not clear, but we know that $x^TAxleqsum{|a_{ij}|}$.



For example:



begin{equation}
A=
begin{bmatrix}
2 & 3 & 1 \
3 & 10 & -8 \
1 & -8 & 17
end{bmatrix}.
end{equation}

begin{equation}
max_{xin{pm1}^n}x^TAx=49<sum |a_{ij}|=53
end{equation}

and in this case $x=[1quad1quad-1]'$ or $x=[-1quad-1quad1]'$. So idea is to sacrifice $2a_{13}$ since it is smaller than $2a_{23}$ and $2a_{12}$.



Is there any systematic way to tell the maximum of $x^TAx$ for any $n$ if $A$ is given? and if yes how to find $x$ that will give the maximum? Any suggestions are welcome :)










share|cite|improve this question
























  • in the example above, spectral radius of $A$ is 22.265 But maximum is 49. Please correct me if I am wrong
    – Lee
    22 hours ago












  • so $|x_i|$=1 and $||x||_{infty}=1$ are similar?
    – Lee
    22 hours ago






  • 1




    what I wanted to say is that all elements of $x$ are equal to 1 or -1, so I think I will better stick to $|x_i|=1$
    – Lee
    22 hours ago












  • @Lee and Rodrigo : Both of you are right. Sorry it is my mistake still, precising that $i$ goes from $1$ to $n$ would not hurt. An alternative way to write is simply $xin{-1,1}^n$
    – Surb
    22 hours ago






  • 1




    This is known as a Binary Quadratic Problem and is hard. There are some solvers for it, for instance www-lipn.univ-paris13.fr/BiqCrunch and you can have some success solving a sequence of semidefinite relaxations.
    – Michal Adamaszek
    20 hours ago















up vote
2
down vote

favorite












Let $A in mathbb{R}^{ntimes n}$ be symmetric and positive definite. What is the following maximum?



$$max_{xin{pm1}^n}x^T A x$$





My attempt:



if all $a_{ij}geq 0$, then



begin{equation}
max_{xin{pm1}^n}x^TAx=sum a_{ij}
end{equation}



and in this case $x=[1quad 1quadcdotsquad 1]'$ or $x=[-1quad -1quadcdotsquad -1]'$.



if some $a_{ij}<0$, then situation is not clear, but we know that $x^TAxleqsum{|a_{ij}|}$.



For example:



begin{equation}
A=
begin{bmatrix}
2 & 3 & 1 \
3 & 10 & -8 \
1 & -8 & 17
end{bmatrix}.
end{equation}

begin{equation}
max_{xin{pm1}^n}x^TAx=49<sum |a_{ij}|=53
end{equation}

and in this case $x=[1quad1quad-1]'$ or $x=[-1quad-1quad1]'$. So idea is to sacrifice $2a_{13}$ since it is smaller than $2a_{23}$ and $2a_{12}$.



Is there any systematic way to tell the maximum of $x^TAx$ for any $n$ if $A$ is given? and if yes how to find $x$ that will give the maximum? Any suggestions are welcome :)










share|cite|improve this question
























  • in the example above, spectral radius of $A$ is 22.265 But maximum is 49. Please correct me if I am wrong
    – Lee
    22 hours ago












  • so $|x_i|$=1 and $||x||_{infty}=1$ are similar?
    – Lee
    22 hours ago






  • 1




    what I wanted to say is that all elements of $x$ are equal to 1 or -1, so I think I will better stick to $|x_i|=1$
    – Lee
    22 hours ago












  • @Lee and Rodrigo : Both of you are right. Sorry it is my mistake still, precising that $i$ goes from $1$ to $n$ would not hurt. An alternative way to write is simply $xin{-1,1}^n$
    – Surb
    22 hours ago






  • 1




    This is known as a Binary Quadratic Problem and is hard. There are some solvers for it, for instance www-lipn.univ-paris13.fr/BiqCrunch and you can have some success solving a sequence of semidefinite relaxations.
    – Michal Adamaszek
    20 hours ago













up vote
2
down vote

favorite









up vote
2
down vote

favorite











Let $A in mathbb{R}^{ntimes n}$ be symmetric and positive definite. What is the following maximum?



$$max_{xin{pm1}^n}x^T A x$$





My attempt:



if all $a_{ij}geq 0$, then



begin{equation}
max_{xin{pm1}^n}x^TAx=sum a_{ij}
end{equation}



and in this case $x=[1quad 1quadcdotsquad 1]'$ or $x=[-1quad -1quadcdotsquad -1]'$.



if some $a_{ij}<0$, then situation is not clear, but we know that $x^TAxleqsum{|a_{ij}|}$.



For example:



begin{equation}
A=
begin{bmatrix}
2 & 3 & 1 \
3 & 10 & -8 \
1 & -8 & 17
end{bmatrix}.
end{equation}

begin{equation}
max_{xin{pm1}^n}x^TAx=49<sum |a_{ij}|=53
end{equation}

and in this case $x=[1quad1quad-1]'$ or $x=[-1quad-1quad1]'$. So idea is to sacrifice $2a_{13}$ since it is smaller than $2a_{23}$ and $2a_{12}$.



Is there any systematic way to tell the maximum of $x^TAx$ for any $n$ if $A$ is given? and if yes how to find $x$ that will give the maximum? Any suggestions are welcome :)










share|cite|improve this question















Let $A in mathbb{R}^{ntimes n}$ be symmetric and positive definite. What is the following maximum?



$$max_{xin{pm1}^n}x^T A x$$





My attempt:



if all $a_{ij}geq 0$, then



begin{equation}
max_{xin{pm1}^n}x^TAx=sum a_{ij}
end{equation}



and in this case $x=[1quad 1quadcdotsquad 1]'$ or $x=[-1quad -1quadcdotsquad -1]'$.



if some $a_{ij}<0$, then situation is not clear, but we know that $x^TAxleqsum{|a_{ij}|}$.



For example:



begin{equation}
A=
begin{bmatrix}
2 & 3 & 1 \
3 & 10 & -8 \
1 & -8 & 17
end{bmatrix}.
end{equation}

begin{equation}
max_{xin{pm1}^n}x^TAx=49<sum |a_{ij}|=53
end{equation}

and in this case $x=[1quad1quad-1]'$ or $x=[-1quad-1quad1]'$. So idea is to sacrifice $2a_{13}$ since it is smaller than $2a_{23}$ and $2a_{12}$.



Is there any systematic way to tell the maximum of $x^TAx$ for any $n$ if $A$ is given? and if yes how to find $x$ that will give the maximum? Any suggestions are welcome :)







optimization positive-definite non-convex-optimization






share|cite|improve this question















share|cite|improve this question













share|cite|improve this question




share|cite|improve this question








edited 22 hours ago

























asked 23 hours ago









Lee

606




606












  • in the example above, spectral radius of $A$ is 22.265 But maximum is 49. Please correct me if I am wrong
    – Lee
    22 hours ago












  • so $|x_i|$=1 and $||x||_{infty}=1$ are similar?
    – Lee
    22 hours ago






  • 1




    what I wanted to say is that all elements of $x$ are equal to 1 or -1, so I think I will better stick to $|x_i|=1$
    – Lee
    22 hours ago












  • @Lee and Rodrigo : Both of you are right. Sorry it is my mistake still, precising that $i$ goes from $1$ to $n$ would not hurt. An alternative way to write is simply $xin{-1,1}^n$
    – Surb
    22 hours ago






  • 1




    This is known as a Binary Quadratic Problem and is hard. There are some solvers for it, for instance www-lipn.univ-paris13.fr/BiqCrunch and you can have some success solving a sequence of semidefinite relaxations.
    – Michal Adamaszek
    20 hours ago


















  • in the example above, spectral radius of $A$ is 22.265 But maximum is 49. Please correct me if I am wrong
    – Lee
    22 hours ago












  • so $|x_i|$=1 and $||x||_{infty}=1$ are similar?
    – Lee
    22 hours ago






  • 1




    what I wanted to say is that all elements of $x$ are equal to 1 or -1, so I think I will better stick to $|x_i|=1$
    – Lee
    22 hours ago












  • @Lee and Rodrigo : Both of you are right. Sorry it is my mistake still, precising that $i$ goes from $1$ to $n$ would not hurt. An alternative way to write is simply $xin{-1,1}^n$
    – Surb
    22 hours ago






  • 1




    This is known as a Binary Quadratic Problem and is hard. There are some solvers for it, for instance www-lipn.univ-paris13.fr/BiqCrunch and you can have some success solving a sequence of semidefinite relaxations.
    – Michal Adamaszek
    20 hours ago
















in the example above, spectral radius of $A$ is 22.265 But maximum is 49. Please correct me if I am wrong
– Lee
22 hours ago






in the example above, spectral radius of $A$ is 22.265 But maximum is 49. Please correct me if I am wrong
– Lee
22 hours ago














so $|x_i|$=1 and $||x||_{infty}=1$ are similar?
– Lee
22 hours ago




so $|x_i|$=1 and $||x||_{infty}=1$ are similar?
– Lee
22 hours ago




1




1




what I wanted to say is that all elements of $x$ are equal to 1 or -1, so I think I will better stick to $|x_i|=1$
– Lee
22 hours ago






what I wanted to say is that all elements of $x$ are equal to 1 or -1, so I think I will better stick to $|x_i|=1$
– Lee
22 hours ago














@Lee and Rodrigo : Both of you are right. Sorry it is my mistake still, precising that $i$ goes from $1$ to $n$ would not hurt. An alternative way to write is simply $xin{-1,1}^n$
– Surb
22 hours ago




@Lee and Rodrigo : Both of you are right. Sorry it is my mistake still, precising that $i$ goes from $1$ to $n$ would not hurt. An alternative way to write is simply $xin{-1,1}^n$
– Surb
22 hours ago




1




1




This is known as a Binary Quadratic Problem and is hard. There are some solvers for it, for instance www-lipn.univ-paris13.fr/BiqCrunch and you can have some success solving a sequence of semidefinite relaxations.
– Michal Adamaszek
20 hours ago




This is known as a Binary Quadratic Problem and is hard. There are some solvers for it, for instance www-lipn.univ-paris13.fr/BiqCrunch and you can have some success solving a sequence of semidefinite relaxations.
– Michal Adamaszek
20 hours ago















active

oldest

votes











Your Answer





StackExchange.ifUsing("editor", function () {
return StackExchange.using("mathjaxEditing", function () {
StackExchange.MarkdownEditor.creationCallbacks.add(function (editor, postfix) {
StackExchange.mathjaxEditing.prepareWmdForMathJax(editor, postfix, [["$", "$"], ["\\(","\\)"]]);
});
});
}, "mathjax-editing");

StackExchange.ready(function() {
var channelOptions = {
tags: "".split(" "),
id: "69"
};
initTagRenderer("".split(" "), "".split(" "), channelOptions);

StackExchange.using("externalEditor", function() {
// Have to fire editor after snippets, if snippets enabled
if (StackExchange.settings.snippets.snippetsEnabled) {
StackExchange.using("snippets", function() {
createEditor();
});
}
else {
createEditor();
}
});

function createEditor() {
StackExchange.prepareEditor({
heartbeatType: 'answer',
convertImagesToLinks: true,
noModals: true,
showLowRepImageUploadWarning: true,
reputationToPostImages: 10,
bindNavPrevention: true,
postfix: "",
imageUploader: {
brandingHtml: "Powered by u003ca class="icon-imgur-white" href="https://imgur.com/"u003eu003c/au003e",
contentPolicyHtml: "User contributions licensed under u003ca href="https://creativecommons.org/licenses/by-sa/3.0/"u003ecc by-sa 3.0 with attribution requiredu003c/au003e u003ca href="https://stackoverflow.com/legal/content-policy"u003e(content policy)u003c/au003e",
allowUrls: true
},
noCode: true, onDemand: true,
discardSelector: ".discard-answer"
,immediatelyShowMarkdownHelp:true
});


}
});














 

draft saved


draft discarded


















StackExchange.ready(
function () {
StackExchange.openid.initPostLogin('.new-post-login', 'https%3a%2f%2fmath.stackexchange.com%2fquestions%2f2999348%2fwhat-is-the-maximum-value-of-xtax-subject-to-x-in-pm1-n%23new-answer', 'question_page');
}
);

Post as a guest















Required, but never shown






























active

oldest

votes













active

oldest

votes









active

oldest

votes






active

oldest

votes
















 

draft saved


draft discarded



















































 


draft saved


draft discarded














StackExchange.ready(
function () {
StackExchange.openid.initPostLogin('.new-post-login', 'https%3a%2f%2fmath.stackexchange.com%2fquestions%2f2999348%2fwhat-is-the-maximum-value-of-xtax-subject-to-x-in-pm1-n%23new-answer', 'question_page');
}
);

Post as a guest















Required, but never shown





















































Required, but never shown














Required, but never shown












Required, but never shown







Required, but never shown

































Required, but never shown














Required, but never shown












Required, but never shown







Required, but never shown







Popular posts from this blog

QoS: MAC-Priority for clients behind a repeater

Ивакино (Тотемский район)

Can't locate Autom4te/ChannelDefs.pm in @INC (when it definitely is there)